Übersicht:

 

25.3 Vektorrechnung - Lösungen

1. Aufgabe

1)
\begin{matrix}-10\vec{a} + 5\vec{b} + \vec{c} &=& -10\begin{pmatrix}-\frac{3}{2}\cr\frac{1}{4}\end{pmatrix} &+& 5\begin{pmatrix}1\cr1\end{pmatrix} &+& \begin{pmatrix}-4\cr-9\end{pmatrix} \cr \cr &=& \begin{pmatrix}-10\cdot \left(-\frac{3}{2}\right)\cr-10\cdot \frac{1}{4}\end{pmatrix} &+& \begin{pmatrix}5\cdot 1\cr5\cdot 1\end{pmatrix} &+& \begin{pmatrix}-4\cr-9\end{pmatrix} \cr \cr &=& \begin{pmatrix}15\cr-\frac{5}{2}\end{pmatrix} &+& \begin{pmatrix}5\cr5\end{pmatrix} &+& \begin{pmatrix}-4\cr-9\end{pmatrix} \cr \cr &=& \begin{pmatrix}16\cr-\frac{13}{2}\end{pmatrix} \end{matrix}


2)
Diese Linearkombination kann nicht berechnet werden, weil die Vektoren nicht die gleiche Anzahl an Komponenten haben.


3)
\begin{matrix}6\vec{a} + \frac{4}{3}\vec{b} - \frac{1}{2}\vec{c} &=& 6\cdot 3\begin{pmatrix}-2\cr-\frac{1}{3}\end{pmatrix} &+& \frac{4}{3}\begin{pmatrix}18\cr-15\end{pmatrix} &-& \frac{1}{2}\begin{pmatrix}-26\cr22\end{pmatrix} \cr \cr &=& \begin{pmatrix}6 \cdot 3 \cdot \left(-2\right)\cr6 \cdot 3 \cdot \left(-\frac{1}{3}\right)\end{pmatrix} &+& \begin{pmatrix}\frac{4}{3} \cdot 18\cr\frac{4}{3} \cdot \left(-15\right)\end{pmatrix} &+& \begin{pmatrix}-\frac{1}{2} \cdot \left(-26\right)\cr-\frac{1}{2} \cdot 22\end{pmatrix} \cr \cr &=& \begin{pmatrix}-36\cr-6\end{pmatrix} &+& \begin{pmatrix}24\cr-20\end{pmatrix} &+& \begin{pmatrix}13\cr-11\end{pmatrix} \cr \cr &=& \begin{pmatrix}1\cr-37\end{pmatrix} \end{matrix}


4)
\begin{matrix}2\vec{a} - 3\vec{b} - 7\vec{c} &=& 2\begin{pmatrix}27\cr-41\end{pmatrix} &-& 3\begin{pmatrix}-25\cr-33\end{pmatrix} &-& 7\left(-\begin{pmatrix}-19\cr31\end{pmatrix}\right) \cr \cr &=& \begin{pmatrix}2\cdot 27\cr2\cdot \left(-41\right)\end{pmatrix} &+& \begin{pmatrix}-3\cdot \left(-25\right)\cr-3\cdot \left(-33\right)\end{pmatrix} &+& \begin{pmatrix}-7\cdot 19\cr-7\cdot \left(-31\right)\end{pmatrix} \cr \cr &=& \begin{pmatrix}54\cr-82\end{pmatrix} &+& \begin{pmatrix}75\cr99\end{pmatrix} &+& \begin{pmatrix}-133\cr217\end{pmatrix} \cr \cr &=& \begin{pmatrix}-4\cr234\end{pmatrix} \end{matrix}


5)
Diese Linearkombination kann nicht berechnet werden, weil die Vektoren nicht die gleiche Anzahl an Komponenten haben.


6)
\begin{matrix}4\vec{a} + 9\vec{b} - 3\vec{c} &=& 4\begin{pmatrix}-16\cr-50\cr-5\end{pmatrix} &+& 9\begin{pmatrix}1\cr-46\cr-6\end{pmatrix} &-& 3\begin{pmatrix}-27\cr33\cr40\end{pmatrix} \cr \cr &=& \begin{pmatrix}4\cdot \left(-16\right)\cr4\cdot \left(-50\right)\cr4\cdot \left(-5\right)\end{pmatrix} &+& \begin{pmatrix}9\cdot 1\cr9\cdot \left(-46\right)\cr9\cdot \left(-6\right)\end{pmatrix} &+& \begin{pmatrix}-3\cdot \left(-27\right)\cr-3\cdot 33\cr-3\cdot 40 \end{pmatrix} \cr \cr &=& \begin{pmatrix}-64\cr-200\cr-20\end{pmatrix} &+& \begin{pmatrix}9\cr-414\cr-54\end{pmatrix} &+& \begin{pmatrix}81\cr-99\cr-120\end{pmatrix} \cr \cr &=& \begin{pmatrix}26\cr-713\cr-194\end{pmatrix} \end{matrix}


7)
\begin{matrix}\vec{v} - 8\vec{w} + 3\vec{u} &=& \begin{pmatrix}38\cr-37\cr24\end{pmatrix} &-& 8\begin{pmatrix}-15\cr36\cr-27\end{pmatrix} &+& 3\begin{pmatrix}-32\cr-2\cr-20\end{pmatrix} \cr \cr &=& \begin{pmatrix}38\cr-37\cr24\end{pmatrix} &+& \begin{pmatrix}-8\cdot \left(-15\right)\cr-8\cdot 36\cr-8\cdot \left(-27\right)\end{pmatrix} &+& \begin{pmatrix}3\cdot \left(-32\right)\cr3\cdot \left(-2\right)\cr3\cdot \left(-20\right)\end{pmatrix} \cr \cr &=& \begin{pmatrix}38\cr-37\cr24\end{pmatrix} &+& \begin{pmatrix}120\cr-288\cr216\end{pmatrix} &+& \begin{pmatrix}-96\cr-6\cr-60\end{pmatrix} \cr \cr &=& \begin{pmatrix}62\cr-331\cr180\end{pmatrix} \end{matrix}


8)
\begin{matrix}-6\vec{a} + 7\vec{b} - 2\vec{c} &=& -6\begin{pmatrix}-4 \cr -20 \cr -17\end{pmatrix} &+& 7\begin{pmatrix}-25\cr-29\cr23\end{pmatrix} &-& 2\begin{pmatrix}16\cr-29\cr-35\end{pmatrix} \cr \cr &=& \begin{pmatrix}-6\cdot \left(-4\right)\cr-6\cdot \left(-20\right)\cr-6\cdot \left(-17\right)\end{pmatrix} &+& \begin{pmatrix}7\cdot \left(-25\right)\cr7\cdot \left(-29\right)\cr7\cdot 23\end{pmatrix} &+& \begin{pmatrix}-2\cdot 16\cr-2\cdot \left(-29\right)\cr-2\cdot \left(-35\right)\end{pmatrix} \cr \cr &=& \begin{pmatrix}24\cr120\cr102\end{pmatrix} &+& \begin{pmatrix}-175\cr-203\cr161\end{pmatrix} &+& \begin{pmatrix}-32\cr58\cr70\end{pmatrix} \cr \cr &=& \begin{pmatrix}-183\cr-25\cr333\end{pmatrix} \end{matrix}


9)
\begin{matrix}\dfrac{5}{6} \vec{t} - 4\vec{s} + \dfrac{7}{2} \vec{q} &=& \dfrac{5}{6} \begin{pmatrix}-27\cr48\cr-38\end{pmatrix} &-& 4\begin{pmatrix}\frac{31}{8}\cr0\cr \frac{1}{12}\end{pmatrix} &+& \dfrac{7}{2} \begin{pmatrix}-44\cr-6\cr \frac{3}{14}\end{pmatrix} \cr \cr &=& \begin{pmatrix}\frac{5}{6} \cdot \left(-27\right)\cr \frac{5}{6} \cdot 48\cr \frac{5}{6} \cdot \left(-38\right)\end{pmatrix} &+& \begin{pmatrix}-4 \cdot \frac{31}{8}\cr-4\cdot 0\cr-4\cdot \frac{1}{12}\end{pmatrix} &+& \begin{pmatrix}\frac{7}{2} \cdot \left(-44\right)\cr \frac{7}{2} \cdot \left(-6\right)\cr \frac{7}{2} \cdot \frac{3}{14}\end{pmatrix} \cr \cr &=& \begin{pmatrix}-\frac{45}{2}\cr40\cr-\frac{95}{3}\end{pmatrix} &+& \begin{pmatrix}-\frac{31}{2}\cr0\cr -\frac{1}{3}\end{pmatrix} &+& \begin{pmatrix}-154\cr-21\cr \frac{3}{4}\end{pmatrix} \cr \cr &=& \begin{pmatrix}-192\cr 19 \cr-\frac{125}{4}\end{pmatrix} \end{matrix}


10)
\begin{matrix}-\dfrac{1}{4} \vec{a} + 12\vec{b} + 9\vec{c} &=& -\dfrac{1}{4}\begin{pmatrix}\frac{16}{3}\cr-8\cr0\end{pmatrix} &+& 12\begin{pmatrix}\frac{19}{4}\cr-2\cr-\frac{17}{2}\end{pmatrix} &+& 9\begin{pmatrix}-\frac{2}{27}\cr\frac{11}{9}\cr-1\end{pmatrix} \cr \cr &=& \begin{pmatrix}-\frac{1}{4}\cdot \frac{16}{3}\cr-\frac{1}{4}\cdot \left(-8\right)\cr-\frac{1}{4} \cdot 0\end{pmatrix} &+& \begin{pmatrix}12\cdot \frac{19}{4}\cr12\cdot \left(-2\right)\cr12\cdot \left(-\frac{17}{2}\right)\end{pmatrix} &+& \begin{pmatrix}9\cdot \left(-\frac{2}{27}\right)\cr9\cdot \frac{11}{9}\cr9\cdot \left(-1\right)\end{pmatrix} \cr \cr &=& \begin{pmatrix}-\frac{4}{3}\cr2\cr0\end{pmatrix} &+& \begin{pmatrix}57\cr-24\cr-102\end{pmatrix} &+& \begin{pmatrix}-\frac{2}{3}\cr11\cr-9\end{pmatrix}\cr \cr &=& \begin{pmatrix}55\cr-11\cr-111\end{pmatrix} \end{matrix}

 

2. Aufgabe

1)
\begin{matrix} \left|\begin{pmatrix}5\cr-14\end{pmatrix}\right| &=& \sqrt{{5}^2+{\left(-14\right)}^2 } \cr &=& \sqrt{221} &\approx & 14{,}87 \end{matrix}


2)
\begin{matrix} \left|\begin{pmatrix}-\frac{3}{10}\cr-\frac{7}{3} \end{pmatrix}\right| &=& \sqrt{{\left(-\dfrac{3}{10}\right)}^2+{\left(-\dfrac{7}{3}\right)}^2 } \cr &=& \sqrt{\dfrac{4981}{900}} &\approx & 2{,}35 \end{matrix}


3)
\begin{matrix} \left|-\begin{pmatrix}0\cr-17\end{pmatrix}\right| &=& \left|\begin{pmatrix}0\cr17\end{pmatrix}\right| \cr \cr &=& \sqrt{{0}^2+{17}^2 } \cr &=& 17 \end{matrix}


4)
\begin{matrix} \left|\begin{pmatrix}8\cr14\cr-17\end{pmatrix}\right| &=& \sqrt{{8}^2+{14}^2+{\left(-17\right)}^2 } \cr &=& \sqrt{549} &\approx & 23{,}43 \end{matrix}


5)
\begin{matrix} \left|\begin{pmatrix}16\cr-12\cr-\frac 27 \end{pmatrix}\right| &=& \sqrt{{16}^2+{(-12)}^2+{\left(-\dfrac{2}{7}\right)}^2 } \cr &=& \sqrt{\dfrac{19604}{49}} &\approx & 20{,}00 \end{matrix}


6)
\begin{matrix} \left|-\dfrac{1}{12}\begin{pmatrix}48\cr-72\cr-12\end{pmatrix}\right| &=& \left|\begin{pmatrix}-4\cr6\cr1\end{pmatrix}\right| \cr \cr &=& \sqrt{{\left(-4\right)}^2+{6}^2+{1}^2 } \cr &=& \sqrt{53} &\approx & 7{,}28 \end{matrix}


7)
\begin{matrix} \left|\dfrac{3}{4} \begin{pmatrix}\frac{8}{3}\cr32\cr16\end{pmatrix}\right| &=& \left|\begin{pmatrix}2\cr24\cr12\end{pmatrix}\right| \cr\cr &=& \sqrt{{2}^2+{24}^2+{12}^2 } \cr &=& \sqrt{724} &\approx & 26{,}91 \end{matrix}


8)
\begin{matrix} \left|\dfrac 25 \begin{pmatrix}-20\cr25\cr-9\end{pmatrix}\right| &=& \left|\begin{pmatrix}-8 \cr10\cr-\frac{18}{5}\end{pmatrix}\right| \cr\cr &=& \sqrt{{\left(-8\right)}^2+{10}^2+{\left(-\dfrac{18}{5}\right)}^2 } \cr &=& \sqrt{\dfrac{4424}{25}} &\approx & 13{,}30 \end{matrix}

 

3. Aufgabe

1)
Für den Betrag gilt:
\begin{array}{rcl} \left|\begin{pmatrix}x\cr-6\cr11\end{pmatrix}\right| &=& \sqrt{{x}^2+{\left(-6\right)}^2 +{11}^2} \cr &=& \sqrt{x^2+157} \end{array}

Zu lösen ist also:
\begin{array}{rclc} \sqrt{x^2+157} &=& 15 \cr x^2+157 &=& 225 \cr x &=& \pm\sqrt{68} &\approx \pm 8{,}25 \end{array}

Ergebnisse:
\left|\begin{pmatrix}8{,}25\cr-6\cr11\end{pmatrix}\right|\approx 15 und \left|\begin{pmatrix}-8{,}25\cr-6\cr11\end{pmatrix}\right|\approx 15


2)
Für den Betrag gilt:
\begin{array}{rcl} \left|\begin{pmatrix}-10\cr2x\cr9\end{pmatrix}\right| &=& \sqrt{{\left(-10\right)}^2+{\left(2x\right)}^2 +{9}^2} \cr &=& \sqrt{4x^2+181} \end{array}

Zu lösen ist also:
\begin{array}{rclc} \sqrt{4x^2+181} &=& 21 \cr 4x^2+181 &=& 441 \cr x &=& \pm\sqrt{65} &\approx \pm 8{,}06 \end{array}

Ergebnisse:
\left|\begin{pmatrix}-10\cr2\cdot 8{,}06\cr9\end{pmatrix}\right|\approx 21 und \left|\begin{pmatrix}-10\cr2\cdot \left(-8{,}06\right)\cr9\end{pmatrix}\right|\approx 21


3)
Für den Betrag gilt:
\begin{array}{rcl} \left|\begin{pmatrix}-4\cr\cr\frac{x}{2}\cr\cr\frac{3x}{2}\end{pmatrix}\right| &=& \sqrt{{\left(-4\right)}^2+{\left(\dfrac{x}{2}\right)}^2 +{\left(\dfrac{3x}{2}\right)}^2} \cr &=& \sqrt{\dfrac{5}{2}x^2+16} \end{array}

Zu lösen ist also:
\begin{array}{rclc} \sqrt{\dfrac{5}{2}x^2+16} &=& 45 \cr \dfrac{5}{2}x^2+16 &=& 2025 \cr x &=& \pm\sqrt{803{,}6} &\approx \pm 28{,}35 \end{array}

Ergebnisse:
\left|\begin{pmatrix}-4\cr\cr\frac{28{,}35}{2}\cr\cr\frac{3\cdot 28{,}35}{2}\end{pmatrix}\right|\approx 45 und \left|\begin{pmatrix}-4\cr\cr\frac{-28{,}35}{2}\cr\cr\frac{3\cdot\left(-28{,}35\right)}{2}\end{pmatrix}\right|\approx 45


4)
Für den Betrag gilt:
\begin{array}{rcl} \left|\begin{pmatrix}-1\cr15\cr12x\end{pmatrix}\right| &=& \sqrt{{\left(-1\right)}^2+{15}^2 +{\left(12x\right)}^2} \cr &=& \sqrt{144x^2+226} \end{array}

Zu lösen ist also:
\begin{array}{rclc} \sqrt{144x^2+226} &=& 28 \cr 144x^2+226 &=& 784 \cr x &=& \pm\sqrt{\dfrac{31}{8}} &\approx \pm 1{,}97 \end{array}

Ergebnisse:
\left|\begin{pmatrix}-1\cr15\cr12\cdot 1{,}97\end{pmatrix}\right|\approx 28 und \left|\begin{pmatrix}-1\cr15\cr12\cdot\left(-1{,}97\right)\end{pmatrix}\right|\approx 28


5)
Für den Betrag gilt:
\begin{array}{rcl} \left|\begin{pmatrix}x\cr-13\cr3x\end{pmatrix}\right| &=& \sqrt{{x}^2+{\left(-13\right)}^2 +{\left(3x\right)}^2} \cr &=& \sqrt{10x^2+169} \end{array}

Zu lösen ist also:
\begin{array}{rclc} \sqrt{10x^2+169} &=& 17 \cr 10x^2+169 &=& 289 \cr x &=& \pm\sqrt{12} &\approx \pm 3{,}46 \end{array}

Ergebnisse:
\left|\begin{pmatrix}3{,}46\cr-13\cr3\cdot 3{,}46\end{pmatrix}\right|\approx 17 und \left|\begin{pmatrix}-3{,}46\cr-13\cr3\cdot\left(-3{,}46\right)\end{pmatrix}\right|\approx 17


6)
Für den Betrag gilt:
\begin{array}{rcl} \left|\begin{pmatrix}-17\cr4x^2\cr14\end{pmatrix}\right| &=& \sqrt{(-17)^2+\left(4x^2\right)^2+14^2} \cr &=& \sqrt{16x^4+485} \end{array}

Zu lösen ist also:
\begin{array}{rclc} \sqrt{16x^4+485} &=& 25 \cr 16x^4+485 &=& 625 \cr x &=& \pm\sqrt[4]{\dfrac{35}{4}} \approx \pm 1{,}72 \end{array}

Ergebnisse:
\left|\begin{pmatrix}-17 \cr 11{,}83 \cr 14\end{pmatrix}\right|\approx 25 und \left|\begin{pmatrix}-17 \cr -11{,}83 \cr 14\end{pmatrix}\right|\approx 25


7)
Für den Betrag gilt:
\begin{array}{rcl}\left|\begin{pmatrix}\sqrt{25}\cr x^3 \cr-\frac{1}{2}\end{pmatrix}\right| &=& \sqrt{\left(\sqrt{25}\right)^2+\left(x^3\right)^2+\left(-\dfrac{1}{2}\right)^2} \cr &=& \sqrt{x^6+\dfrac{101}{4}} \end{array}

Zu lösen ist also:
\begin{array}{rclc} \sqrt{x^6+\dfrac{101}{4}} &=& 11 \cr x^6+\dfrac{101}{4} &=& 121 \cr x &=& \pm\sqrt[6]{\dfrac{383}{4}} \approx \pm 2{,}14 \end{array}

Ergebnisse:
\left|\begin{pmatrix}\sqrt{25} \cr 9{,}79 \cr -\frac{1}{2}\end{pmatrix}\right|\approx 11 und \left|\begin{pmatrix}\sqrt{25} \cr -9{,}79 \cr -\frac{1}{2}\end{pmatrix}\right|\approx 11


8)
Für den Betrag gilt:
\begin{array}{rcl} \left|\begin{pmatrix}\sqrt{x^3}\cr 12 \cr 12\end{pmatrix}\right| &=& \sqrt{\left(\sqrt{x^3}\right)^2+12^2+12^2} \cr &=& \sqrt{x^3+288} \end{array}

Zu lösen ist also:
\begin{array}{rclc} \sqrt{x^3+288} &=& \sqrt{504} \cr x^3+288 &=& 504 \cr x &=& 6 \end{array}

Ergebnis:
\left|\begin{pmatrix}\sqrt{6^3} \cr 12 \cr 12\end{pmatrix}\right| = \sqrt{504}

 

4. Aufgabe

Bemerkung: Bitte vergessen Sie bei allen diesen Aufgaben das \pm vor dem Ergebnis nicht. Es geht ja schließlich um den Betrag von \lambda...


1)
\begin{array}{rcl} \left|\lambda\begin{pmatrix}7\cr-8\cr10\end{pmatrix}\right| &=& \sqrt{{\left(7\lambda\right)}^2+{\left(-8\lambda\right)}^2 +{\left(10\lambda\right)}^2} \cr &=& \sqrt{\lambda^2 \left(49+64+100\right)} \cr &=& \left|\lambda\right|\sqrt{213} \end{array}

Zu lösen ist also:
\begin{array}{rclc} \left|\lambda\right|\sqrt{213} &=& 1 \cr \lambda &=& \pm\dfrac{1}{\sqrt{213}} &\approx \pm 0{,}07 \end{array}

Ergebnisse:
\left|0{,}07\begin{pmatrix}7\cr-8\cr10\end{pmatrix}\right|\approx 1 und \left|-0{,}07\begin{pmatrix}7\cr-8\cr10\end{pmatrix}\right|\approx 1


2)
\begin{array}{rcl} \left|\lambda\begin{pmatrix}-\frac{4}{9}\cr0\cr\frac{1}{3}\end{pmatrix}\right| &=& \sqrt{{\left(-\dfrac{4}{9}\lambda\right)}^2+{\left(0\lambda\right)}^2 +{\left(\dfrac{1}{3}\lambda\right)}^2} \cr &=& \sqrt{\lambda^2 \left(\dfrac{16}{81}+\dfrac{1}{9}\right)} \cr &=& \left|\lambda\right|\sqrt{\dfrac{25}{81}} \cr &=& \left|\lambda\right|\dfrac{5}{9}\end{array}

Zu lösen ist also:
\begin{array}{rclc} \left|\lambda\right|\dfrac{5}{9} &=& 1 \cr \lambda &=& \pm\dfrac{9}{5}\end{array}

Ergebnisse:
\left|\dfrac{9}{5}\begin{pmatrix}-\frac{4}{9}\cr0\cr\frac{1}{3}\end{pmatrix}\right|=1 und \left|-\dfrac{9}{5}\begin{pmatrix}-\frac{4}{9}\cr0\cr\frac{1}{3}\end{pmatrix}\right|=1


3)
\begin{array}{rcl} \left|\lambda\begin{pmatrix}-40\cr-20\cr70\end{pmatrix}\right| &=& \sqrt{{\left(-40\lambda\right)}^2+{\left(-20\lambda\right)}^2 +{\left(70\lambda\right)}^2} \cr &=& \sqrt{\lambda^2 \left(1.600+400+4.900\right)} \cr &=& \left|\lambda\right|\sqrt{6.900} \end{array}

Zu lösen ist also:
\begin{array}{rclc} \left|\lambda\right|\sqrt{6.900} &=& 1 \cr \lambda &=& \pm\dfrac{1}{\sqrt{6.900}} &\approx \pm 0{,}01 \end{array}

Ergebnisse:
\left|0{,}01\begin{pmatrix}-40\cr-20\cr70\end{pmatrix}\right|\approx 1 und \left|-0{,}01\begin{pmatrix}-40\cr-20\cr70\end{pmatrix}\right|\approx 1


4)
\begin{array}{rcl} \left|\lambda\begin{pmatrix}23\cr-35\cr-27\end{pmatrix}\right| &=& \sqrt{{\left(23\lambda\right)}^2+{\left(-35\lambda\right)}^2 +{\left(-27\lambda\right)}^2} \cr &=& \sqrt{\lambda^2 \left(529+1.225+729\right)} \cr &=& \left|\lambda\right|\sqrt{2.483} \end{array}

Zu lösen ist also:
\begin{array}{rclc} \left|\lambda\right|\sqrt{2.483} &=& 1 \cr \lambda &=& \pm\dfrac{1}{\sqrt{2.483}} &\approx \pm 0{,}02 \end{array}

Ergebnisse:
\left|0{,}02\begin{pmatrix}23\cr-35\cr-27\end{pmatrix}\right|\approx 1 und \left|-0{,}02\begin{pmatrix}23\cr-35\cr-27\end{pmatrix}\right|\approx 1


5)
\begin{array}{rcl} \left|\lambda\begin{pmatrix}3\cr7\cr-1\end{pmatrix}\right| &=& \sqrt{{\left(3\lambda\right)}^2+{\left(7\lambda\right)}^2 +{\left(-\lambda\right)}^2} \cr &=& \sqrt{\lambda^2 \left(9+49+1\right)} \cr &=& \left|\lambda\right|\sqrt{59} \end{array}

Zu lösen ist also:
\begin{array}{rclc} \left|\lambda\right|\sqrt{59} &=& 1 \cr \lambda &=& \pm\dfrac{1}{\sqrt{59}} &\approx \pm 0{,}13 \end{array}

Ergebnisse:
\left|0{,}13\begin{pmatrix}3\cr7\cr-1\end{pmatrix}\right|\approx 1 und \left|-0{,}13\begin{pmatrix}3\cr7\cr-1\end{pmatrix}\right|\approx 1


6)
\begin{array}{rcl} \left|\lambda\begin{pmatrix}-64\cr 14 \cr -10 \end{pmatrix}\right| &=& \sqrt{(-64\lambda)^2+(14\lambda)^2+(-10\lambda)^2} \cr &=& \sqrt{\lambda^2 \left(4.096+196+100\right)} \cr &=& \left|\lambda\right|\sqrt{4.392} \end{array}

Zu lösen ist also:
\begin{array}{rclc} \left|\lambda\right|\sqrt{4.392} &=& 1 \cr \lambda &=& \pm\dfrac{1}{\sqrt{4.392}} &\approx \pm 0{,}02 \end{array}

Ergebnisse:
\left|0{,}02\begin{pmatrix}-64\cr 14 \cr -10 \end{pmatrix}\right|\approx 1 und \left|-0{,}02\begin{pmatrix}-64\cr 14 \cr -10 \end{pmatrix}\right|\approx 1


7)
\begin{array}{rcl} \left|\lambda\left(-\dfrac{4}{5}\right) \begin{pmatrix}15 \cr -\frac{5}{12} \cr 0\end{pmatrix}\right| &=& \sqrt{\left(-\dfrac{4}{5} \cdot 15 \lambda\right)^2 + \left(-\dfrac{4}{5}\cdot\left(-\dfrac{5}{12}\right)\lambda\right)^2+ \left(-\dfrac{4}{5} \cdot 0\cdot \lambda\right)^2} \cr &=& \sqrt{\lambda^2 \left(144+ \dfrac{1}{9} +0\right)} \cr &=& \left|\lambda\right|\sqrt{\dfrac{1.297}{9}} \end{array}

Zu lösen ist also:
\begin{array}{rclc} \left|\lambda\right|\sqrt{\dfrac{1.297}{9}} &=& 1 \cr \lambda &=& \pm\dfrac{1}{\sqrt{\frac{1.297}{9}}} &\approx \pm 0{,}08 \end{array}

Ergebnisse:
\left|0{,}08\cdot\left(-\dfrac{4}{5}\right) \begin{pmatrix}15 \cr -\frac{5}{12} \cr 0\end{pmatrix}\right|\approx 1 und \left|-0{,}08\cdot\left(-\dfrac{4}{5}\right) \begin{pmatrix}15 \cr -\frac{5}{12} \cr 0\end{pmatrix}\right|\approx 1


8)
\begin{array}{rcl} \left|\lambda\begin{pmatrix}-\sqrt{20}\cr 13 \cr 25 \end{pmatrix}\right| &=&\sqrt{\lambda^2 \left(-\sqrt{20}\right)^2+13^2+25^2} \cr &=& \sqrt{\lambda^2 \left(20+169+625\right)} \cr &=& \left|\lambda\right|\sqrt{814} \end{array}

Zu lösen ist also:
\begin{array}{rclc} \left|\lambda\right|\sqrt{814} &=& 1 \cr \lambda &=& \pm\dfrac{1}{\sqrt{814}} &\approx \pm 0{,}04 \end{array}

Ergebnisse:
\left|0{,}04\begin{pmatrix}-\sqrt{20}\cr 13 \cr 25 \end{pmatrix}\right|\approx 1 und \left|-0{,}04\begin{pmatrix}-\sqrt{20}\cr 13 \cr 25 \end{pmatrix}\right|\approx 1

 

5. Aufgabe

1)
\vec{a}^{\,T} \cdot \vec{b} = 43 \cdot 2 + \left(-45\right) \cdot \left(-3\right) + 11 \cdot \left(-4\right) = 177


2)
\vec{a}^{\,T} \cdot \vec{b} = -\dfrac{5}{8} \cdot \dfrac{4}{5} \cdot 24 + \left(-13\right) \cdot \dfrac{4}{5} \cdot 30 + 20 \cdot \dfrac{4}{5} \cdot 7 = -212


3)
\vec{a}^{\,T} \cdot \vec{b} = 12 \cdot \left(-8\right) + 14 \cdot 12 + \left(-20\right) \cdot \left(-18\right) = 432


4)
\vec{x}^{\,T} \cdot \vec{y} = -\dfrac{3}{2} \cdot 4 + 0 \cdot \left(-7\right) + 36 \cdot \dfrac{1}{6} = 0


5)
\vec{a}^{\,T} \cdot \vec{b} = 121 \cdot \left(-84\right) + \left(-97\right) \cdot \left(-113\right) + 105 \cdot 96 = 10.877


6)
\vec{a}^{\,T} \cdot \vec{b} = \dfrac{4}{3} \cdot 3+13 \cdot (-37)+16 \cdot \dfrac{121}{4} = 7


7)
\vec{a}^{\,T} \cdot \vec{b} = \dfrac{3}{5} \cdot \dfrac{8}{3} \cdot -30 + -28 \cdot \dfrac{8}{3} \cdot 0 + \dfrac{1}{3} \cdot \dfrac{8}{3} \cdot 0 = -48


8)
\vec{a}^{\,T} \cdot \vec{b} = 3t \cdot 225 + (-9) \cdot (-256) + (-14) \cdot 289 = 675t-1.742

 

6. Aufgabe

Wichtig: Denken Sie daran, Ihren Taschenrechner auf Gradmaß einzustellen!


1)
{\varphi = \arccos \left( \dfrac{ 4 \cdot \left(-1\right) + 4 \cdot \left(-9\right) + \left(-6\right) \cdot \left(-5\right)}{\sqrt{{4}^2+{4}^2+{\left(-6\right)}^2} \cdot \sqrt{{\left(-1\right)}^2+{\left(-9\right)}^2+{\left(-5\right)}^2}} \right) = \arccos \left( \dfrac{-10}{\sqrt{68}\cdot\sqrt{107}} \right) \approx 96{,}73^{\circ}}


2)
{\varphi = \arccos \left( \dfrac{ 2 \cdot \left(-1\right) + 2 \cdot 10 + 1 \cdot \left(-1\right)}{\sqrt{{2}^2+{2}^2+{1}^2} \cdot \sqrt{{\left(-1\right)}^2+{10}^2+{\left(-1\right)}^2}} \right) = \arccos \left( \dfrac{17}{\sqrt{9}\cdot\sqrt{102}} \right) \approx 55{,}87^{\circ}}


3)
{\varphi = \arccos \left( \dfrac{ -17 \cdot 2 + \left(-7\right) \cdot 28 + \left(-8\right) \cdot 9}{\sqrt{{\left(-17\right)}^2+{\left(-7\right)}^2+{\left(-8\right)}^2} \cdot \sqrt{{2}^2+{28}^2+{9}^2}} \right) = \arccos \left( \dfrac{-302}{\sqrt{402}\cdot\sqrt{869}} \right) \approx 120{,}73^{\circ}}


4)
{\varphi = \arccos \left( \dfrac{ \frac{5}{4} \cdot \left(-\frac{8}{15}\right) + 9 \cdot \left(-\frac{4}{3}\right) + \frac{7}{2} \cdot (-10)}{\sqrt{{\left(\frac{5}{4}\right)}^2+{9}^2+{\left(\frac{7}{2}\right)}^2} \cdot \sqrt{{\left(-\frac{8}{15}\right)}^2+{\left(-\frac{4}{3}\right)}^2+{\left(-10\right)}^2}} \right) = \arccos \left( \dfrac{-\frac{143}{3}}{\sqrt{\frac{1.517}{16}}\cdot\sqrt{\frac{22.964}{225}}} \right) \approx 118{,}98^{\circ}}


5)
{\varphi = \arccos \left( \dfrac{ -15 \cdot \frac{22}{5} + \left(-\frac 14\right) \cdot 12 + 18 \cdot \left(-\frac{7}{2}\right)}{\sqrt{{(-15)}^2+{\left(-\frac 14\right)}^2+{18}^2} \cdot \sqrt{{\left(\frac{22}{5}\right)}^2+{12}^2+{\left(-\frac{7}{2}\right)}^2}} \right) = \arccos \left( \dfrac{-132}{\sqrt{\frac{8.785}{16}}\cdot\sqrt{\frac{17.561}{100}}} \right) \approx 115{,}16^{\circ}}


6)
{\varphi = \arccos \left(\dfrac{\frac{27}{11}\cdot 22 + \sqrt{3}\cdot\sqrt{12} + 8\cdot 16}{\sqrt{\left(\frac{27}{11}\right)^2+\left(\sqrt{3}\right)^2+8^2} \cdot \sqrt{22^2+ \left(\sqrt{12}\right)^2+16^2}}\right) = \arccos \left(\dfrac{188}{\sqrt{\frac{8.836}{121}} \cdot \sqrt{752}}\right) \approx 36{,}65^{\circ}}


7)
{\varphi = \arccos \left(\dfrac{-\frac{5}{6} \cdot 3 + \sqrt{8} \cdot \left(-\sqrt{2}\right) + 0 \cdot 5^3}{\sqrt{\left(-\frac{5}{6}\right)^2+\left(\sqrt{8}\right)^2+0^2} \cdot \sqrt{3^2+ \left(-\sqrt{2}\right)^2+\left(5^3\right)^2}} \right) = \arccos \left(\dfrac{-\frac{13}{2}}{\sqrt{\frac{313}{36}} \cdot \sqrt{15.636}} \right) \approx 91{,}01^{\circ}}


8)
{\varphi = \arccos \left(\dfrac{-2^7 \cdot\left(-\frac{1}{2} \right) + \left(-7^3\right) \cdot 0 + 0 \cdot \sqrt{15}}{\sqrt{\left(-2^7\right)^2+\left(-7^3\right)^2+0^2} \cdot \sqrt{\left(-\frac{1}{2}\right)^2+ 0^2+\left(\sqrt{15}\right)^2}} \right) = \arccos \left(\dfrac{64}{\sqrt{134.033} \cdot \sqrt{15{,}25}} \right) \approx 87{,}43^{\circ}}

 

7. Aufgabe

1)
\vec{a}^{\,T} \cdot \vec{b} = 1 \cdot 18 + 0 \cdot \left(-10\right) + \left(-3\right) \cdot 6 = 0
Die Vektoren sind also orthogonal.


2)
\vec{a}^{\,T} \cdot \vec{b} = \dfrac{1}{2} \cdot \left(-16\right) + 1 \cdot 15 + 5 \cdot \dfrac{11}{10} = 12{,}5
Die Vektoren sind also nicht orthogonal.


3)
\vec{DE}^{\,T} \cdot \vec{FG} = 13 \cdot 2 + \left(-10\right) \cdot 2 + 3 \cdot \left(-2\right) = 0
Die Vektoren sind also orthogonal.


4)
\vec{a}^{\,T} \cdot \vec{b} = 25 \cdot 3\cdot 4 + \left(-30\right) \cdot 3 \cdot \left(-3\right) + \left(-34\right) \cdot 3\cdot 6 = -42
Die Vektoren sind also nicht orthogonal.


5)
\vec{a}^{\,T} \cdot \vec{b} = 9 \cdot 0 + \left(-7\right) \cdot \left(-\dfrac{9}{14}\right) + 0 \cdot \dfrac{8}{3} = \dfrac{9}{2}
Die Vektoren sind also nicht orthogonal.


6)
\vec{a}^{\,T} \cdot \vec{b} = \sqrt{32} \cdot \dfrac{5}{2} +z \cdot 0+(-2{,}5) \cdot 4 \sqrt{2} = 0
Die Vektoren sind also orthogonal.


7)
\vec{0F}^{\,T} \cdot \vec{w} = 16 \cdot \dfrac{3}{16} + \dfrac{3}{16} \cdot 4 + 3 \cdot 2 = \dfrac{39}{4}
Die Vektoren sind also nicht orthogonal.


8)
\vec{a}^{\,T} \cdot \vec{b} = 18 \cdot 16 + (-8) \cdot 117 + 27 \cdot 24 = 0
Die Vektoren sind also orthogonal.

 

8. Aufgabe

Eine Bemerkung vorab: Das Lösen der auftretenden Linearen Gleichungssysteme wird hier nur recht knapp behandelt. Wer Schwierigkeiten damit hat, schaue im Kapitel 7 nach.


1)
\vec{a} = \begin{pmatrix}-12-\left(-45\right) \cr -36-15\end{pmatrix} = \begin{pmatrix}33 \cr -51\end{pmatrix} und \vec{b} = \begin{pmatrix}-78-\left(-12\right) \cr 66-\left(-36\right)\end{pmatrix} = \begin{pmatrix}-66 \cr 102\end{pmatrix}

Die Gleichung \lambda_1\begin{pmatrix}33 \cr -51\end{pmatrix} + \lambda_2\begin{pmatrix}-66 \cr 102\end{pmatrix} = \vec{0} führt zu folgendem Gleichungssystem:

\begin{array}{crclllcc} \text{I} & 33\lambda_1 - 66\lambda_2 &=& 0 & \vert \cdot 51 \cr \text{II} & -51\lambda_1 + 102\lambda_2 &=& 0 & \vert \cdot 33 \cr \cr \text{I} & 33\lambda_1 - 66\lambda_2 &=& 0 \cr \text{5 I+33 II} & 0\lambda_1 + 0\lambda_2 &=& 0 \end{array}

Das Gleichungssystem hat also unendlich viele Lösungen, z. B. \lambda_1 =1 und \lambda_2 =2 (dies kann man entweder durch scharfes Hinsehen herausfinden oder indem man beide Gleichungen nach \lambda_1 auflöst). Die Vektoren sind also linear abhängig.


2)
\vec{a} = \begin{pmatrix}-15-\left(-19\right) \cr 14-30\end{pmatrix} = \begin{pmatrix}4 \cr -16\end{pmatrix} und \vec{b} = \begin{pmatrix}-10-\left(-15\right) \cr 0-14\end{pmatrix} = \begin{pmatrix}5 \cr -14\end{pmatrix}

Die Gleichung \lambda_1\begin{pmatrix}4 \cr -16\end{pmatrix} + \lambda_2\begin{pmatrix}5 \cr -14\end{pmatrix} = \vec{0} führt zu folgendem Gleichungssystem:

\begin{array}{crclllcc} \text{I} & 4\lambda_1 + 5\lambda_2 &=& 0 & \vert \cdot 4 \cr \text{II} & -16\lambda_1 - 14\lambda_2 &=& 0 \cr \cr \text{I} & 4\lambda_1 + 5\lambda_2 &=& 0 \cr \text{4 I+II} & 0\lambda_1 + 6\lambda_2 &=& 0 & \Rightarrow \quad \lambda_2 = 0 \quad \Rightarrow \quad \lambda_1 = 0\end{array}

Das Gleichungssystem hat nur die Lösung \lambda_1=\lambda_2=0 . Die Vektoren sind also linear unabhängig.


3)
\vec{a} = \begin{pmatrix}-\frac{5}{3} - \frac{10}{3}\cr\frac{71}{8} - \left(-\frac{49}{8}\right)\cr-\frac{1}{10} - \left(-\frac{651}{10}\right)\end{pmatrix} = \begin{pmatrix}-5\cr15\cr65\end{pmatrix} , \vec{b} = \begin{pmatrix}-\frac{4}{3} -\left(-\frac{5}{3}\right)\cr\frac{7}{8} - \frac{71}{8}\cr-\frac{1}{10} - \left(-\frac{1}{10}\right)\end{pmatrix} = \begin{pmatrix}\frac{1}{3}\cr-8\cr0\end{pmatrix} und \vec{c} = \begin{pmatrix}-\frac{1}{3} - \left(-\frac{4}{3}\right)\cr\frac{7}{8} - \frac{7}{8}\cr-\frac{11}{10} - \left(-\frac{1}{10}\right)\end{pmatrix} = \begin{pmatrix}1\cr0\cr-1\end{pmatrix}

Die Gleichung \lambda_1\begin{pmatrix}-5\cr15\cr65\end{pmatrix} + \lambda_2\begin{pmatrix}\frac{1}{3}\cr-8\cr0\end{pmatrix} + \lambda_3\begin{pmatrix}1\cr0\cr-1\end{pmatrix} = \vec{0} führt zu folgendem Gleichungssystem:

\begin{array}{crcrcrcrl} \text{I} & -5\lambda_1 &+& \dfrac{1}{3}\lambda_2 &+& \lambda_3 &=& 0 \cr \text{II} & 15\lambda_1 &-& 8\lambda_2 & & &=& 0 \cr \text{III} & 65\lambda_1 & & &-& \lambda_3 &=& 0 \cr \cr \text{I} & -5\lambda_1 &+& \dfrac{1}{3}\lambda_2 &+& \lambda_3 &=& 0 \cr \text{3 I+II} & & & -7\lambda_2 &+& 3\lambda_3 &=& 0 \cr \text{13 I+III} & & & \dfrac{13}{3}\lambda_2 &+& 12\lambda_3 &=& 0 \cr \cr \text{I} & -5\lambda_1 &+& \dfrac{1}{3}\lambda_2 &+& \lambda_3 &=& 0 \cr \text{II'} & & & -7\lambda_2 &+& 3\lambda_3 &=& 0 \cr \text{-4 II'+III'} & & & \dfrac{97}{3}\lambda_2 & & &=& 0 & \Rightarrow \quad \lambda_2 = 0 \quad \Rightarrow \quad \lambda_3 = 0 \quad \Rightarrow \quad \lambda_1 = 0 \end{array}

Das Gleichungssystem hat nur die Lösung \lambda_1=\lambda_2=\lambda_3=0 . Die Vektoren sind also linear unabhängig.


4)
\vec{a} = \begin{pmatrix}15 - 33\cr-1 - \left(-7\right)\cr5 - \left(-19\right)\end{pmatrix} = \begin{pmatrix}-18\cr6\cr24\end{pmatrix} , \vec{b} = \begin{pmatrix}17 - 15\cr-3 - \left(-1\right)\cr-9 - 5\end{pmatrix} = \begin{pmatrix}2\cr-2\cr-14\end{pmatrix} und \vec{c} = \begin{pmatrix}1 - 17\cr1 - \left(-3\right)\cr1 - (-9)\end{pmatrix} = \begin{pmatrix}-16\cr4\cr10\end{pmatrix}

Die Gleichung \lambda_1\begin{pmatrix}-18\cr6\cr24\end{pmatrix} + \lambda_2\begin{pmatrix}2\cr-2\cr-14\end{pmatrix} + \lambda_3\begin{pmatrix}-16\cr4\cr10\end{pmatrix} = \vec{0} führt zu folgendem Gleichungssystem:

\begin{array}{crcrcrcrl} \text{I} & -18\lambda_1 &+& 2\lambda_2 &-& 16\lambda_3 &=& 0 \cr \text{II} & 6\lambda_1 &-& 2\lambda_2 &+& 4\lambda_3 &=& 0 \cr \text{III} & 24\lambda_1 &-& 14\lambda_2 &+& 10\lambda_3 &=& 0 \cr \cr \text{I} & -18\lambda_1 &+& 2\lambda_2 &-& 16\lambda_3 &=& 0 \cr \text{I+3 II} & & & -4\lambda_2 &-& 4\lambda_3 &=& 0 \cr \text{4 I+3 III} & & & -34\lambda_2 &-& 34\lambda_3 &=& 0 \cr \cr \text{I} & -18\lambda_1 &+& 2\lambda_2 &-& 16\lambda_3 &=& 0 \cr \text{II'} & & & -4\lambda_2 &-& 4\lambda_3 &=& 0 \cr \text{-17 II'+2 III'} & & & 0\lambda_2 &+& 0\lambda_3 &=& 0 \end{array}

Das Gleichungssystem hat also unendlich viele Lösungen, z. B. \lambda_1 =1 , \lambda_2 =1 und \lambda_3=-1 (dies kann man entweder durch scharfes Hinsehen herausfinden oder indem man z. B. für \lambda_3 einen Wert wählt und damit \lambda_2 und \lambda_1 berechnet). Die Vektoren sind also linear abhängig.


5)
\vec{a} = \begin{pmatrix}20 - 19\cr20 - 21\cr-2 - \left(-3\right)\end{pmatrix} = \begin{pmatrix}1\cr-1\cr1\end{pmatrix} , \vec{b} = \begin{pmatrix}10 - 20\cr15 - 20\cr-9 - \left(-2\right)\end{pmatrix} = \begin{pmatrix}-10\cr-5\cr-7\end{pmatrix} und \vec{c} = \begin{pmatrix}20 - 10\cr13 - 15\cr-7 - \left(-9\right)\end{pmatrix} = \begin{pmatrix}10\cr-2\cr2\end{pmatrix}

Die Gleichung \lambda_1\begin{pmatrix}1\cr-1\cr1\end{pmatrix} + \lambda_2\begin{pmatrix}-10\cr-5\cr-7\end{pmatrix} + \lambda_3\begin{pmatrix}10\cr-2\cr2\end{pmatrix} = \vec{0} führt zu folgendem Gleichungssystem:

\begin{array}{crcrcrcrl} \text{I} & \lambda_1 &-& 10\lambda_2 &+& 10\lambda_3 &=& 0 \cr \text{II} & -\lambda_1 &-& 5\lambda_2 &-& 2\lambda_3 &=& 0 \cr \text{III} & \lambda_1 &-& 7\lambda_2 &+& 2\lambda_3 &=& 0 \cr \cr \text{I} & \lambda_1 &-& 10\lambda_2 &+& 10\lambda_3 &=& 0 \cr \text{I+II} & & & -15\lambda_2 &+& 8\lambda_3 &=& 0 \cr \text{I-III} & & & -3\lambda_2 &+& 8\lambda_3 &=& 0 \cr \cr \text{I} & \lambda_1 &-& 10\lambda_2 &+& 10\lambda_3 &=& 0 \cr \text{II'} & & & -15\lambda_2 &+& 8\lambda_3 &=& 0 \cr \text{II'-5 III'} & & & & & -32\lambda_3 &=& 0 & \Rightarrow \quad \lambda_3 = 0 \quad \Rightarrow \quad \lambda_2 = 0 \quad \Rightarrow \quad \lambda_1 = 0 \end{array}

Das Gleichungssystem hat nur die Lösung \lambda_1=\lambda_2=\lambda_3=0 . Die Vektoren sind also linear unabhängig.

 

9. Aufgabe

1)
\vec{a} = \begin{pmatrix}14-\left(-1\right) \cr -5-0 \cr -2-\left(-8\right)\end{pmatrix} = \begin{pmatrix}15 \cr -5 \cr 6\end{pmatrix} und \vec{b} = \begin{pmatrix}-31-14 \cr 10-\left(-5\right) \cr -20-\left(-2\right) \end{pmatrix} = \begin{pmatrix}-45 \cr 15 \cr -18\end{pmatrix}

Da -3\begin{pmatrix}15 \cr -5 \cr 6\end{pmatrix} = \begin{pmatrix}-45 \cr 15 \cr -18\end{pmatrix} gilt, sind die beiden Vektoren antiparallel.


2)
\vec{a} = \begin{pmatrix}-18-\left(-11\right) \cr \frac{16}{5}-\frac{11}{5} \cr -\frac{17}{2}-\left(-7\right)\end{pmatrix} = \begin{pmatrix}-7 \cr 1 \cr -\frac{3}{2}\end{pmatrix} und \vec{b} = \begin{pmatrix}24-\left(-18\right) \cr -\frac{14}{5}-\frac{16}{5} \cr \frac{1}{2}-\left(-\frac{17}{2}\right)\end{pmatrix} = \begin{pmatrix}42 \cr -6 \cr 9\end{pmatrix}

Da -6\begin{pmatrix}-7 \cr 1 \cr -\frac{3}{2}\end{pmatrix} = \begin{pmatrix}42 \cr -6 \cr 9\end{pmatrix} gilt, sind die beiden Vektoren antiparallel.


3)
\vec{a} = \begin{pmatrix}2-1 \cr 7-6 \cr -8-\left(-16\right)\end{pmatrix} = \begin{pmatrix}1 \cr 1 \cr 8\end{pmatrix} und \vec{b} = \begin{pmatrix}-2-2 \cr 2-7 \cr -40-\left(-8\right)\end{pmatrix} = \begin{pmatrix}-4 \cr -5 \cr -32\end{pmatrix}

Da es keine Zahl \lambda gibt, für die \lambda\begin{pmatrix}1 \cr 1 \cr 8\end{pmatrix} = \begin{pmatrix}-4 \cr -5 \cr -32\end{pmatrix} gilt, sind die beiden Vektoren weder parallel noch antiparallel.


4)
\vec{a} = \begin{pmatrix}12-8 \cr 2-\left(-9\right) \cr 0-10\end{pmatrix} = \begin{pmatrix}4 \cr 11 \cr -10\end{pmatrix} und \vec{b} = \begin{pmatrix}24-12 \cr 35-2 \cr -30-0\end{pmatrix} = \begin{pmatrix}12 \cr 33 \cr -30\end{pmatrix}

Da 3\begin{pmatrix}4 \cr 11 \cr -10\end{pmatrix} = \begin{pmatrix}12 \cr 33 \cr -30\end{pmatrix} gilt, sind die beiden Vektoren parallel.


5)
\vec{a} = \begin{pmatrix}-12-\left(-9\right) \cr 9-12 \cr -23-\left(-31\right)\end{pmatrix} = \begin{pmatrix}-3 \cr -3 \cr 8\end{pmatrix} und \vec{b} = \begin{pmatrix}-39-\left(-12\right) \cr -18-9 \cr 49-\left(-23\right)\end{pmatrix} = \begin{pmatrix}-27 \cr -27 \cr 72\end{pmatrix}

Da 9\begin{pmatrix}-3 \cr -3 \cr 8\end{pmatrix} = \begin{pmatrix}-27 \cr -27 \cr 72\end{pmatrix} gilt, sind die beiden Vektoren parallel.


6)
\vec{a} = \begin{pmatrix}14-14 \cr 3-\left(-4\right) \cr 21-7\end{pmatrix} = \begin{pmatrix}0 \cr 7\cr 14\end{pmatrix} und \vec{b} = \begin{pmatrix}14-14 \cr 0-3 \cr 20-21\end{pmatrix} = \begin{pmatrix}0 \cr -3 \cr -1\end{pmatrix}

Da es keine Zahl \lambda gibt, für die \lambda\begin{pmatrix}0 \cr 7\cr 14\end{pmatrix} = \begin{pmatrix}0 \cr -3 \cr -1\end{pmatrix} gilt, sind die beiden Vektoren weder parallel noch antiparallel.

 

10. Aufgabe

1)
\vec{a} \times \vec{b} = \begin{pmatrix}\frac{4}{3} \cdot \left(-\frac{9}{2}\right) &-& (-12) \cdot \frac{5}{6} \cr -12 \cdot \left(-\frac{3}{4}\right) &-& 0 \cdot \left(-\frac{9}{2}\right) \cr 0 \cdot \frac{5}{6} &-& \frac{4}{3} \cdot \left(-\frac{3}{4}\right)\end{pmatrix} = \begin{pmatrix}4\cr9\cr1\end{pmatrix}


2)
\vec{n} = \frac{3}{4} \begin{pmatrix}-4\cr12\cr-32\end{pmatrix} = \begin{pmatrix}-3\cr9\cr-24\end{pmatrix}

\vec{o} \times \vec{n} = \begin{pmatrix}-7 \cdot \left(-24\right) &-& 6 \cdot 9 \cr 6 \cdot \left(-3\right) &-& 14 \cdot \left(-24\right) \cr 14 \cdot 9 &-& \left(-7\right) \cdot \left(-3\right)\end{pmatrix} = \begin{pmatrix}114\cr318\cr105\end{pmatrix}


3)
\vec{a} \times \vec{b} = \begin{pmatrix}-45 \cdot 11 &-& \left(-43\right) \cdot 13 \cr -43 \cdot 9 &-& \left(-35\right) \cdot 11 \cr -35 \cdot 13 &-& \left(-45\right) \cdot 9\end{pmatrix} = \begin{pmatrix}64\cr-2\cr-50\end{pmatrix}


4)
\vec{a} \times \vec{b} = \begin{pmatrix}4 \cdot \frac{8}{5} &-& \left(-12\right) \cdot 11 \cr -12 \cdot \left(-\frac{4}{3}\right) &-& \frac {13}{6} \cdot \frac{8}{5} \cr \frac{13}{6} \cdot 11 &-& 4 \cdot \left(-\frac{4}{3}\right)\end{pmatrix} = \begin{pmatrix}\frac{692}{5} \cr \frac{188}{15} \cr \frac{175}{6}\end{pmatrix} \approx \begin{pmatrix}138{,}4 \cr 12{,}53 \cr 29{,}17\end{pmatrix}


5)
\vec{a} = -\frac{5}{4} \begin{pmatrix}22\cr-13\cr-40\end{pmatrix} = \begin{pmatrix}-\frac{55}{2}\cr \frac{65}{4}\cr50\end{pmatrix} und \vec{b} = \frac{4}{5} \begin{pmatrix}0\cr-30\cr4\end{pmatrix} = \begin{pmatrix}0\cr-24\cr \frac{16}{5}\end{pmatrix}

\vec{a} \times \vec{b} = \begin{pmatrix}\frac{65}{4} \cdot \frac{16}{5} &-& 50 \cdot \left(-24\right) \cr 50 \cdot 0 &-& \left(-\frac{55}{2} \right)\cdot \frac{16}{5} \cr -\frac{55}{2} \cdot (-24) &-& \frac{65}{4} \cdot 0\end{pmatrix} = \begin{pmatrix}1.252\cr88\cr660\end{pmatrix}


6)
\vec{a} \times \vec{b} = \begin{pmatrix}\frac{1}{2} \cdot 84 &-& 14 \cdot 3 \cr 14 \cdot 6 &-& \left(-\frac{3}{2}\right) \cdot 84\cr -\frac{3}{2}\cdot 3 &-& \frac{1}{2} \cdot 6\end{pmatrix} = \begin{pmatrix}0 \cr 210 \cr -\frac{15}{2} \end{pmatrix}


7)
\vec{a} \times \vec{b} = \begin{pmatrix}0 \cdot 0 &-& \frac{13}{2} \cdot \left(-\frac{15}{9}\right) \cr \frac{13}{2} \cdot 0 &-& \left(-\frac{11}{72}x\right) \cdot 0 \cr -\frac{11}{72}x \cdot \left(-\frac{15}{9}\right) &-& 0 \cdot 0\end{pmatrix} = \begin{pmatrix} \frac{65}{6} \cr 0 \cr \frac{55}{216}x \end{pmatrix}


8)
\vec{a} \times \vec{b} = \begin{pmatrix}17 \cdot (-66) &-& 0 \cdot 10 \cr 0 \cdot \left(-4\right) &-& \left(-\frac{3}{5}\right) \cdot (-66) \cr \left(-\frac{3}{5}\right) \cdot 10 &-& 17 \cdot \left(-4\right)\end{pmatrix} = \begin{pmatrix}-1.122 \cr -\frac{198}{5} \cr 62 \end{pmatrix}

 

11. Aufgabe

1)
\begin{matrix}\vec{a}^{\,T} \cdot \left(\vec{b} \times \vec{c}\right) &=& \begin{pmatrix}1;\,-1;\,2\end{pmatrix} \cdot \left(\begin{pmatrix}10\cr0\cr-8\end{pmatrix} \times \begin{pmatrix}4\cr-5\cr0\end{pmatrix}\right) \cr \cr &=& \begin{pmatrix}1;\,-1;\,2\end{pmatrix} \cdot \begin{pmatrix}0 \cdot 0 &-& (-8) \cdot (-5) \cr -8 \cdot 4 &-& 10 \cdot 0 \cr 10 \cdot (-5) &-& 0 \cdot 4 \end{pmatrix} \cr \cr &=& \begin{pmatrix}1;\,-1;\,2\end{pmatrix} \cdot \begin{pmatrix}-40\cr-32\cr-50\end{pmatrix} \cr \cr &=& 1 \cdot \left(-40\right) + \left(-1\right) \cdot \left(-32\right) + 2 \cdot \left(-50\right) &=& -108\end{matrix}


2)
\vec{c} = \frac{1}{3} \begin{pmatrix}-21\cr18\cr48\end{pmatrix} = \begin{pmatrix}-7\cr6\cr16\end{pmatrix}

\begin{matrix}\vec{a}^{\,T} \cdot \left(\vec{b} \times \vec{c}\right) &=& \begin{pmatrix}-21;\,6;\,-35\end{pmatrix} \cdot \left(\begin{pmatrix}-7\cr-18\cr28\end{pmatrix} \times \begin{pmatrix}-7\cr6\cr16\end{pmatrix}\right) \cr \cr &=& \begin{pmatrix}-21;\,6;\,-35\end{pmatrix} \cdot \begin{pmatrix}-18 \cdot 16 &-& 28 \cdot 6 \cr 28 \cdot (-7) &-& (-7) \cdot 16 \cr -7 \cdot 6 &-& (-18) \cdot (-7) \end{pmatrix} \cr \cr &=& \begin{pmatrix}-21;\,6;\,-35\end{pmatrix} \cdot \begin{pmatrix}-456\cr-84\cr-168\end{pmatrix} \cr \cr &=& -21 \cdot \left(-456\right) + 6 \cdot \left(-84\right) + \left(-35\right) \cdot \left(-168\right) &=& 14.952\end{matrix}


3)
\begin{matrix}\vec{a}^{\,T} \cdot \left(\vec{b} \times \vec{c}\right) &=& \begin{pmatrix}37;\,7;\,-32\end{pmatrix} \cdot \left(\begin{pmatrix}-30\cr-5\cr13\end{pmatrix} \times \begin{pmatrix}-10\cr-45\cr-48\end{pmatrix}\right) \cr \cr &=& \begin{pmatrix}37;\,7;\,-32\end{pmatrix} \cdot \begin{pmatrix}-5 \cdot (-48) &-& 13 \cdot (-45) \cr 13 \cdot (-10) &-& (-30) \cdot (-48) \cr -30 \cdot (-45) &-& (-5) \cdot (-10)\end{pmatrix} \cr \cr &=& \begin{pmatrix}37;\,7;\,-32\end{pmatrix} \cdot \begin{pmatrix}825\cr-1.570\cr1.300\end{pmatrix} \cr \cr &=& 37 \cdot 825 + 7 \cdot \left(-1.570\right) + \left(-32\right) \cdot 1.300 &=& -22.065\end{matrix}


4)
\vec{b} = \dfrac{12}{5} \begin{pmatrix}-15\cr1\cr-\frac{15}{4}\end{pmatrix} = \begin{pmatrix}-36\cr\frac{12}{5}\cr-9\end{pmatrix}

\begin{matrix}\vec{a}^{\,T} \cdot \left(\vec{b} \times \vec{c}\right) &=& \begin{pmatrix}-25;\,4;\,1\end{pmatrix} \cdot \left(\begin{pmatrix}-36\cr\frac{12}{5}\cr-9\end{pmatrix} \times \begin{pmatrix}-10\cr11\cr\frac{3}{2}\end{pmatrix}\right) \cr \cr &=& \begin{pmatrix}-25;\,4;\,1\end{pmatrix} \cdot \begin{pmatrix}\frac{12}{5} \cdot \frac{3}{2} &-& (-9) \cdot 11 \cr -9 \cdot (-10) &-& (-36) \cdot \frac{3}{2} \cr -36 \cdot 11 &-& \frac{12}{5} \cdot (-10)\end{pmatrix} \cr \cr &=& \begin{pmatrix}-25;\,4;\,1\end{pmatrix} \cdot \begin{pmatrix}\frac{513}{5}\cr144\cr-372\end{pmatrix} \cr \cr &=& -25 \cdot \dfrac{513}{5} + 4 \cdot 144 + 1 \cdot \left(-372\right) &=& -2.361\end{matrix}


5)
\begin{matrix}\vec{e}^{\,T} \cdot \left(\vec{f} \times \vec{g}\right) &=& \begin{pmatrix}-12;\,2;\,-3\end{pmatrix} \cdot \left(\begin{pmatrix}7\cr-9\cr-9\end{pmatrix} \times \begin{pmatrix}-6\cr-10\cr-14\end{pmatrix}\right) \cr \cr &=& \begin{pmatrix}-12;\,2;\,-3\end{pmatrix} \cdot \begin{pmatrix}-9 \cdot (-14) &-& (-9) \cdot (-10) \cr -9 \cdot (-6) &-& 7 \cdot (-14) \cr 7 \cdot (-10) &-& (-9) \cdot (-6)\end{pmatrix} \cr \cr &=& \begin{pmatrix}-12;\,2;\,-3\end{pmatrix} \cdot \begin{pmatrix}36\cr152\cr-124\end{pmatrix} \cr \cr &=& -12 \cdot 36 + 2\cdot 152 + \left(-3\right) \cdot \left(-124\right) &=& 244\end{matrix}